SIE Exam Question 983: Answer and Explanation

Question: 983

A stock with a beta of -1.5 and a standard deviation of 10.1 will change in which of the following ways if the stock market increases 8%?

  • A. Increase by 4%
  • B. Increase by 8%
  • C. Decrease by 10%
  • D. Decrease by 12%

Correct Answer: D

Explanation:

8% × -1.5 = -12%

A stock with a beta of -1.5 will move 150% in the opposite direction of the market. Therefore, if the stock market increases by 8%, the stock will decrease by 12%.

All content of site and practice tests © 2022 Jack.
Quick View

FINRA Practice Tests